The top of a tree makes angles s and t with points K and L on the ground, respectively, such that the angles are complementary. Point K is x meters and point L is y meters from the base of the tree.

1. In terms of x and y, find the height of the tree. Include your calculations.
2. If m∠t = 40° and y = 8 meters, calculate the height of the tree, rounded to two decimal places.

The Top Of A Tree Makes Angles S And T With Points K And L On The Ground, Respectively, Such That The

Answers

Answer 1

1. In terms of x and y, the height of the tree is [tex]h = y \times tan \ t^\circ[/tex] and[tex]h = x \times tan \ s^\circ[/tex]

2. The height of the tree is 6.71 meters

Trigonometry

From the question, we are to determine the height of the tree in terms of x and y

Using SOH CAH TOA, we can write that

[tex]tan \ t^\circ =\frac{h}{y}[/tex]

∴ [tex]h = y \times tan \ t^\circ[/tex]

Also,

[tex]tan \ s^\circ =\frac{h}{x}[/tex]

∴ [tex]h = x \times tan \ s^\circ[/tex]

2. If m∠t = 40° and y = 8 meters,

Then, the height of the tree

h = 8 × tan40°

h = 6.71 meters

Hence, the height of the tree is 6.71 meters

Lear more on Trigonometry here: https://brainly.com/question/15977788

#SPJ1


Related Questions

Use the grouping method to factor x³ + x² + 3x+3.
A. (x²+1)(x+3)
B. x(x+3)(x + 1)
C. (x + 1)(x+3)
D. (x + 1)(x2+3)

Answers

Answer:

D. [tex](x + 1)(x^2 + 3)[/tex]

Step-by-step explanation:

Hello!

We can group the first two terms and the last two terms.

Factor by Grouping[tex]x^2 + x^2 + 3x + 3[/tex][tex]x^2(x + 1) + 3(x + 1)[/tex][tex](x^2 + 3)(x + 1)[/tex]

Factoring by grouping is the process of breaking down larger polynomials to smaller ones to factor. We can then combine like factors.

In the second step, we can see that we can rewrite [tex]x^3 + x^2[/tex] as [tex]x^2(x + 1)[/tex], as both the two terms share a common factor of [tex]x^2[/tex]. We can pull out [tex]x^2[/tex] from that expression. Similarly, [tex]3x[/tex] and [tex]3[/tex] share a common factor of [tex]3[/tex], so we can pull that out.

Click an item in the list or group of pictures at the bottom of the problem and, holding the button down, drag it into the correct position in the answer box. Release your mouse button when the item is place. If you change your mind, drag the item to the trashcan. Click the trashcan to clear all your answers.
One leg of a right triangle is 2 inches and the hypotenuse is 6 inches.

Find the area of the triangle.

_[tex]\sqrt{x} \\[/tex]

Answers

We can see that the area of the triangle is: 5.65in².

What is a triangle?

A triangle is actually known to be a shape that has three sides, three angles and three vertices. It's also known as a plane shape.

In order to find the area of the triangle, we will use Pythagorean Theorem: c² = a² + b².

Where c = hypotenuse = 6in.

b = 2in.

6² = a² + 2²

36 = a² + 4

a² = 36 - 4 = 32

a = √32

Area of the triangle = ½ × base × height.

Where base = √32

height = 2

Thus: A = ½ × √32 × 2

A = √32 = 5.65in².

Learn more about area of triangle on https://brainly.com/question/17335144

#SPJ1

What are the missing parts that correctly complete the proof?


Drag the answers into the boxes to correctly complete the proof.

(Please refer to the images provided for the answer options and equation image.)

Answers

It has been proved that point A is equidistant from the sides of angle PQR.

What is a Bisector ?

Any line or point that divides an angle or a side in equal parts is called a Bisector.

It is given that

Point A is the bisector of Angle PQR

Referring to the image

1. QA is the bisector of angle PQR : Given

2.Angle PQA = Angle RQA : Definition of Bisector

3.Angle QXA = Angle QYA : Definitions of Perpendicular

4. Angle QXA = Angle QYA : All right angles are congruent.

5. QA ≅QA :  Reflexive Property of Congruence

6. ΔPQA ≅ΔRQA : AAS Congruence Postulate

7. AX ≅AY  Corresponding parts of Congruent Triangles

8. Point A is equidistant from the sides of Angle PQR : Definition of equidistant.

To know more about Bisector

https://brainly.com/question/12896755

#SPJ1

Situation:
A 45 gram sample of a substance that's
used to sterilize surgical instruments has
a k-value of 0.15.
N = Noe kt
No initial mass (at time t = 0)
N= mass at time t
k= a positive constant that depends on
the substance itself and on the units
used to measure time
t-time, in days
Find the substance's half-life, in days.
Round your answer to the nearest tenth.
Enter the correct answer.
DONE
+?

Answers

The substance's half-life is 4.7 days if the 11-gram sample of a substance that’s used to treat thyroid disorders has a k- the value of 0.15

What is exponential decay?

During exponential decay, a quantity falls slowly at first before rapidly decreasing. The exponential decay formula is used to calculate population decline and can also be used to calculate half-life.

We have an exponential function:

[tex]\rm N = N_oe^{-kt}[/tex]

Plug N = N⁰/2

[tex]\rm N_o/2 = N_oe^{-kt}[/tex]

[tex]\rm \dfrac{1}{2} = e^{-0.15t}[/tex]

Solving for t:

t = 4.62≈ 4.7 days

Thus, the substance's half-life is 4.7 days if the 11-gram sample of a substance that’s used to treat thyroid disorders has a k- the value of 0.15

Learn more about exponential decay here:

brainly.com/question/14355665

#SPJ1

what 1 - 2/9 - 1/3 - 1/6 =

Answers

Answer:

5/18

Step-by-step explanation:

The LCD of 9, 3, and 6 is 18.

1 - 2/9 - 1/3 - 1/6 =

= 18/18 - 4/18 - 6/18 - 3/18

= 5/18

Answer: 5/18

Step-by-step explanation:

1/1 - 2/9 - 1/3 - 1/6

18 - 4 - 6 - 3/18

5/18

please help guys how do i order fractions for least to greatest and greatest to least itll nean so much to me for my finals tmrw

Answers

Answer:

If the denominators are equal for all of the fractions, you can simply look at the numerator and order them. If the denominators aren't equal, you'll have to find the lcm (lowest common multiple), for example, if we have the fractions 3/6 and 1/4, we would find the lcm of 6 and 4, which is 12 and multiply the numerators accordingly. So the fractions would then become 6/12 and 3/12, this makes it easier for you to figure out their order from least to greatest.

Goodluck for your finals :)

Which of the following statements is false?

Answers

In quadrilateral PQRS, ∠P and ∠S are opposite angles.

The question is the image below.

Answers

Answer:

Hence - 7 is the answer

Pls mark me brainliest plss

Can someone help me please and thxxx

Answers

Answer:

C

Step-by-step explanation:

cos 30=12/x

[tex]x = 8 \sqrt{3} [/tex]

cos60=y/x

[tex]y =4 \sqrt{3} [/tex]

Answer:

x = [tex]8\sqrt{3}[/tex]

y = [tex]4\sqrt{3}[/tex]

Step-by-step explanation:

• tan 60° = 12/ y         [tan θ = opposite/ adjacent]

y = 12/ tan 60°  

y = [tex]4\sqrt{3}[/tex]

• x² = y² + 12²             [Pythagoras's theorem]

x² = ( [tex]4\sqrt{3}[/tex] )² + 144

x = [tex]\sqrt{(4\sqrt{3})^{2} \space\ + \space\ 144}[/tex]

x = [tex]8\sqrt{3}[/tex]

Probability and Two-Way Tables

Answers

Answer:

the answer are A and Y , B and X.

 A and X are two independent events  , Option A is the correct answer.

When is Probability ?

Probability is the likelihood of an event to happen

A two way table is given

The two events will be independent when

If A is the first event and B is the second event , then

P(A) = P(A|B)

Here the P(A) = 30/100 = 0.3

P(B) = 20/100 = 0.2

P(A|X) = 15/50 = 0.3

P(A) = P(A|X)

Therefore A and X are two independent events , Option A is the correct answer.

To know more about Probability

https://brainly.com/question/11234923

#SPJ1

Un cartero reparte 10.700 cartas al mes. estimar cunats cartas reparte en un semestre

Answers

Teniendo en cuenta la regla de tres simple, el cartero reparte 64.200 cartas en un semestre.

Regla de tres

La regla de tres es una forma de resolver problemas de proporcionalidad entre tres valores conocidos y un valor desconocido, estableciendo una relación de proporcionalidad entre todos ellos.

Es decir, lo que se pretende con ella es encontrar el cuarto término de una proporción conociendo los otros tres.

Si la relación entre las magnitudes es directa, es decir, cuando una magnitud aumenta, la otra también (o cuando una magnitud disminuye, la otra también), se debe aplicar la regla de tres directa.

Para resolver una regla de tres directa se debe seguir la siguiente fórmula, siendo a, b y c datos conocidos y x la variable a calcular:

a ⇒ b

c ⇒ x

Entonces: [tex]x=\frac{cxb}{a}[/tex]

Cartas repartidas en un semestre

En este caso, se puede aplicar la regla de tres de la siguiente manera: Entonces, si el cartero en un mes reparte 10.700 cartas, ¿en 6 meses (un semestre) reparte cuántas cartas?

1 mes ⇒ 10.700 cartas

6 meses ⇒ ×

Entonces: [tex]cantidad de cartas=\frac{6 mesesx10.700 cartas}{1 mes}[/tex]

Resolviendo:

cantidad de cartas= 64.200 cartas

En resumen, el cartero reparte 64.200 cartas en un semestre.

Aprende más con estos ejemplos:

https://brainly.com/question/21685920

https://brainly.com/question/24786153

#SPJ1

HELP PLSSSSSS hdghgdhj

Answers

Answer:

A

Step-by-step explanation:

Use the Pythagorean Theorem: a^2 + b^2 = c^2

The cable will be the hypotenuse, so c = 11.3 m

One leg is the distance of the anchor, so a = 4.2 m

(4.2)^2 + b^2 = (11.3)^2

17.64 + b^2 = 127.69

Subtract 17.64 from both sides

b^2 = 110.05

Square root both sides

b = ~10.5 m

what is the length of the base? help me please thank u ;)​

Answers

Answer:   26

=============================================================

Explanation:

a = 26 and b = 26 are the congruent legs

angle C = 60 degrees is between sides 'a' and b

c = unknown base which is opposite angle C

We'll use the law of cosines to find this missing side.

c^2 = a^2 + b^2 - 2*a*b*cos(C)

c^2 = 26^2 + 26^2 - 2*26*26*cos(60)

c^2 = 676

c = sqrt(676)

c = 26

It turns out that all three sides are the same length (26), which means this isosceles triangle is really equilateral. Consequently, it means all three interior angles are 60 degrees each.

---------------

Here's another way to see why we have an equilateral triangle.

The vertex angle is 60 degrees. Let x be the measure of each base angle. Those two base angles add to the 60 degrees to get 180

x+x+60 = 180

2x+60 = 180

2x = 180-60

2x = 120

x = 120/2

x = 60

Each base angle is 60 degrees, so all three angles are 60 degrees. This points us to the triangle being equilateral and we can say all three sides are 26 units long.

If we didn't have an equilateral triangle, then we'd have no choice but to use the law of cosines mentioned earlier.

[tex]\quad \huge \quad \quad \boxed{ \tt \:Answer }[/tex]

[tex]\qquad \tt \rightarrow \:base \:\:side = 26 \:\: units [/tex]

____________________________________

[tex] \large \tt Solution \: : [/tex]

As the two sides of the triangle are equal, the corresponding angles opposite to the the sides are equal as well.

[tex] \textsf{let each of those angles measure ' x ' } [/tex]

[tex]\qquad \tt \rightarrow \:x + x + 60 = 180[/tex]

[ sum of all interior angles of a triangle ]

[tex]\qquad \tt \rightarrow \:2x + 60 = 180[/tex]

[tex]\qquad \tt \rightarrow \:2x = 180 - 60[/tex]

[tex]\qquad \tt \rightarrow \:2x = 120[/tex]

[tex]\qquad \tt \rightarrow \:x = \cfrac{120}{2} [/tex]

[tex]\qquad \tt \rightarrow \:x = 60 \degree[/tex]

Therefore, all angles of the triangle are equal. that being the case we can conclude that it's an equilateral triangle.

henceforth, all its sides are equal to one another.

[tex]\qquad \tt \rightarrow \:base \: \: side = \: \: 26 \: \: units[/tex]

Given the following data points, calculate the curve of best fit. show all steps.

Answers

Based on the calculations, the equation for the curve of best fit is equal to y = -30.17x + 14.49.

How to calculate the curve of best fit?

From the table of data points, we have the following:

∑x = 16∑y = 50.9∑xy = 24.6∑x² = 35

Mathematically, the standard equation of a straight line is given by:

y = ax + b       ....equation 1.

Thus, the equations that can be used to model the given data points are:

∑y = na + b∑x             ....equation 2.

∑xy = a∑x + b∑x²       ....equation 3.

Substituting the parameters into the equations, we have;

50.9 = 6a + 16b             ....equation 4.

24.6 = 16a + 35b       ....equation 5.

Solving eqn. 5 and 6 simultaneously, we have:

a = -30.17.b = 14.49.

Substituting the value of a and b into eqn. 1, we have;

y = ax + b

y = -30.17x + 14.49.

Therefore, the equation for the curve of best fit is equal to y = -30.17x + 14.49.

Read more on curve of best fit here: brainly.com/question/9366563

#SPJ1

EMERGENCY!! 25 POINTS!!
Make a table for the following data.

a) English, Maths, Urdu, Science, English, Urdu, Science, Maths, English, Computer, Maths, Urdu, Science, Maths, Computer, English.

Answers

Using tally marks make a table of the given subjects English (4), Urdu (3), Maths (4), Science (3) and Computer (2).

The given data are English, Maths, Urdu, Science, English, Urdu, Science, Maths, English, Computer, Maths, Urdu, Science, Maths, Computer, and English.

What is a data table?

A table is an arrangement of information or data, typically in rows and columns, or possibly in a more complex structure. Tables are widely used in communication, research, and data analysis.

The data table for the given data is given below:

Thus, using tally marks make a table of the given subjects English (4), Urdu (3), Maths (4), Science (3) and Computer (2).

To learn more about the data table visit:

https://brainly.com/question/17084863.

#SPJ1

Answer:

huh

Step-by-step explanation:

The lengths represented by BE, EC, and CD on the diagram were determined to be 1800 feet, 200 feet, and 500 feet,
What is the length, in feet, across the lake?
Fill in the blank by entering only a number as your answer.

Answers

The length of lake is 4500 feet.

What is Triangle?

A triangle is a closed shape with 3 angles, 3 sides, and 3 vertices. A triangle with three vertices P, Q, and R is represented as △PQR.

Here, In ΔABE and ΔDCE

          ∠ABE = ∠DCE         { each 90⁰ }

          ∠AEB = ∠CED         { vertically opposite angle}

By AA similarity, we get

          ΔABE ≈ ΔDCE

Now, AB/CD = BE/CE

       AB/500 = 1800/200

       AB/500 = 9

        AB = 9 X 500

       AB = 4500 feet.

Thus, the length of lake is 4500 feet.

Learn more about Triangle from:

https://brainly.com/question/2773823

#SPJ1

Katrina drinks 0.5 gallons of water per day. Which expression shows how to find the number of cups of water she drinks in a week?

There are 16 cups in a gallon.

Answers

Answer:

D

Hope this helps!

A sign on a roadway at the top of a mountain indicates that for the next 4 miles, the grade is 10.5°. Find the change in elevation over that distance for a car descending the mountain. Round to the nearest hundredth.

Answers

the change in elevation is -0.73 miles (the negative sign is because the new elevation is smaller than the initial one).

How to find the change in elevation?

To do this, we can think on the situation as a right triangle, where the hypotenuse is 4 miles, the angle that we look at measures 10.5°, and the change in elevation (let's call it x) will be the opposite cathetus to that angle.

Then we can use the relation:

Sin(a) = (opposite cathetus)/(hypotenuse)

Replacing what we know, we get:

sin(10.5°) = x/4mi

x = sin(10.5°)*4mi = 0.73mi

So the change in elevation is -0.73 miles (the negative sign is because the new elevation is smaller than the initial one).

If you want to learn more about right triangles:

https://brainly.com/question/2217700

#SPJ1

Help needed
a hunid points :)

Answers

Answer:

the answer is C, 5a^6b

Step-by-step explanation:

the bearing of two points x and y from z are 45° and 135° respectively . if |zx|=8cm and |zy|=6cm, find |xy|.

Answers

Answer:

[tex]|{\sf XY}| = 10\; {\rm cm}[/tex].

Step-by-step explanation:

Refer to the diagram attached. The dashed segment attached to [tex]\!{\sf Z}[/tex] points to the north. Rotating this segment clockwise with point [tex]{\sf Z}\!\![/tex] as the fixed center of rotation would eventually align this segment with the one between point [tex]\!\!{\sf Z}[/tex] and point [tex]\!\!{\sf X}[/tex]. The bearing of point [tex]{\sf X}[/tex] from point [tex]{\sf Z}[/tex] is the size of the angle between these two line segments when measured in the clockwise direction.

Subtract the bearing of [tex]{\sf Y}[/tex] from [tex]{\sf Z}[/tex] from the bearing of [tex]{\sf X}[/tex] from [tex]{\sf Z}[/tex] to find the measure of the angle [tex]\angle {\sf YZX}[/tex]:

[tex]\begin{aligned}\angle {\sf YZX} &= 135^{\circ} - 45^{\circ} \\ &= 90^{\circ}\end{aligned}[/tex].

Thus, triangle [tex]\triangle {\sf YZX}[/tex] is a right triangle ([tex]90^{\circ}[/tex]) with segment [tex]{\sf YX}[/tex] as the hypotenuse. It is given that [tex]|{\sf XZ}| = 6\; {\rm cm}[/tex] whereas [tex]|{\sf ZY}| = 6\; {\rm cm}[/tex]. Thus, by Pythagorean's Theorem:

[tex]\begin{aligned}|{\sf ZY}| &= \sqrt{|{\sf ZX}|^{2} + |{\sf ZY}|^{2}} \\ &= \sqrt{(8\; {\rm cm})^{2} + (6\; {\rm cm})^{2}} \\ &= 10\; {\rm cm}\end{aligned}[/tex].

Help please asap points an brainlist

Answers

(X,y) —> slope of line
M —> given point on the line
(X1,y1)—> any point on line

Here's an equation of a line:
y - 4x = 9
Which of the following is the equation of a line
that is parallel?
Click on the correct answer.
-4y+ x = 18
y + 4x = 4
2y - 8x = 3
4y + x = 18

Answers

Answer:

[tex]2y - 8x = 3[/tex]

Step-by-step explanation:

[tex]y - 4x = 9[/tex]

[tex]2y - 8x = 18[/tex]

So 2y - 8x = 3 is parallel to y - 4x = 9.

a-3/7 devided by 3-a/21

Answers

The answer is pretty simple but I put it in a picture below

bag contains one red pen, four black pens, and three blue pens. Two pens are randomly chosen from the bag and
are not replaced.
To the nearest hundredth, what is the probability that a black pen IS chosen first and then another black pen is
chosen?

Answers

ANSWER AT THE BOTTOM

Question:

There is a bag with 1 red pen, 4 black pens, and 3 blue pens. If 2 pens are chosen from the bag without replacement, what is the probability that you chose 2 black pens.

Explanation:

Right now there are a total of 8 pens. 4 of them are black. So, the probability of choosing a black pen right now is 4/8, or 1/2.

Lets assume we picked a pen and got black.

Now there are only 7 pens in the bag, and only 3 of them are black. The probability of choosing a black pen right now is 3/7.

So on the first draw, the probability is 1/2

And on the second draw, the probability is 3/7

To find the probability of 2 ocurrences happening in a row, we must multiply their individual probabilities.

For example, if we wanted to find the probability of rolling two 6's in a row on a dice, we would need to mutiply the individual probability together. The probability of rolling one 6 is 1/6, so the probability of rolling two 6's in a row is 1/6 MULTIPLIED BY 1/6, which is 1/36.

The probability of rolling two 6's in a row is 1/36.

Lets apply the same principle to our situation right now.

So on the first draw, the probability is 1/2

And on the second draw, the probability is 3/7

1/2 MULTIPLIED BY 3/7    =    3/14

3/14 in decimal form is 0.21

ANSWER:

0.21, OR 21%

32 out of 40 kids said they prefer summer over winter. 25 out of 32 adults said they prefer summer. Did a larger percentage of kids or adults prefer summer.

Answers

Answer:

Larger percentage of kids

Step-by-step explanation:

Kids: 32/40 = 4/5

Adults: 25/32 = 5/8

4/5 > 5/8

(6 + 7) + 8 = 21

Choose an equivalent addition sentence that shows the associative property of addition.

Answers

Answer:

6 + (7 + 8) = 21

Step-by-step explanation:

the associative property of addition is: (a + b) + c = a + (b + c)

So the equivalent addition sentence would be:

(6 + 7) + 8 = 6 + (7 + 8) as they both equal 21

What is the sum? startfraction 2 over x squared endfraction startfraction 4 over x squared endfraction

Answers

Answer:

  6/x²

Step-by-step explanation:

Rational expressions are added the same way numerical fractions are added. Their sum is the quotient of the sum of numerators, and the common denominator.

__

fraction sum

The given fractions already have the same denominator, so we simply add their numerators.

  [tex]\dfrac{2}{x^2}+\dfrac{4}{x^2}=\dfrac{2+4}{x^2}=\boxed{\dfrac{6}{x^2}}[/tex]

1 − 2 + 3 −... + 99
Please solve!

Answers

Answer:

50

Step-by-step explanation:

So you can think of this by grouping it like this:

(1-2) + (3-4) + (5-6) + ... + (97-98) + 99

which is equal to: (-1) + (-1) + (-1)... + (-1) + 99

(each group is equal to -1, and 99 won't have a pair since it's the last one)

then, find how many groups of -1 there are:

the groups start at 1 and end at 98, but there are two in each group, so 98/2 = 49. this means there are 49 groups.

so now, you know that there are 49 -1s, so 49 * (-1) = -49.

finally, you can't forget the extra 99 that didn't have a pair, so -49 + 99 = 50.

Which function is graphed?

Answers

Answer:

science type of food group : fibre

Need help please. Math and I'll give 30 points. Please and thank you. :D

And have a great day! :)

Answers

Answer:

h = 17.5 m

Step-by-step explanation:

We have 2 similar right triangles as models of the situation

one has legs with George's height and shadow

the other is the height h of the tree and length of it's shadow

the corresponding sides of the 2 triangles are in proportion, that is

[tex]\frac{6.25}{h}[/tex] = [tex]\frac{10}{28}[/tex] ( cross- multiply )

10h = 175 ( divide both sides by 10 )

h = 17.5 m

Other Questions
Find the value of x.5X11y Match the following.1. to favor or support2. to found or settle a territory3. to recognize or identify by examination4. to construct5. a safe place6. to overrun or inhabit7. cleverness; originality8. permanent residentinhabitantingenuityinfesterectadvocatediagnosehavencolonize what were The postives of colonial rule of Africans A client has been diagnosed with systolic heart failure. what percentage will the nurse expect the patients ejection fraction to be? Six 20 watt, 12-volt luminaires (lighting fixtures) are supplied from a small, 12-volt transformer that is 60 feet away from the fixtures. the wire used is 14 awg. what is the voltage applied to the fixtures? use k = 11.6. A decrease in the autonomic tone of the smooth muscle in a blood vessel would result in. ich topics and guidelines fall into four main categories: Es ridculo que los gobiernos no (hacer) nada para parar el calentamiento global. What is the difference between a proprietorship and a joint-venture company? please help mee, sketch the graph please (50 points will give brainliest!!!) Let f(x)=4x and g(x)=4x+12. Which transformations are needed to transform the graph of f(x) to the graph of g(x)? Use the drop-down menus to complete the statements. Navigating your way home through an unfamiliar route due to road construction would draw upon your ________ intelligence. What is the slope intercept of the line below 04. Describe one way in which decay by microorganisms is useful. The online promotional method that involves applying specific Web design and development techniques to Web pages to increase the possibility that the pages will appear near the top of a search results list is called ____. In a study of cell phone usage and brain hemispheric dominance, an Internet survey was e-mailed to 6965 subjects randomly selected from an online group involved with ears. There were 1302 surveys returned. Use a 0.01 significance level to test the claim that the return rate is less than 20%. Use the P-value method and use the normal distribution as an approximation to the binomial distribution A can of coke which is 12.0ml and is at a temperature of 18.0C is placed into a refrigerator freezer. Thecan was accidently left over night and the temperature dropped to -1.00C. What would happen to thevolume of the coke can? Q2:- Which branch of chemistry deal with the structure and properties of naturally occurring molecules? (c) Nicole has a part-time job entering data in a computer. Her pay at the end of the week Cis affected by the amount of data she enters each hour. What is the dependentvariable? Jessica received a $70 gift card for a coffee store. She used it in buying some coffee that cost $7.26 per pound. After buying the coffee, she had $48.22 left on her card. How many pounds of coffee did she buy?